Solutions To Assignment-3: September 19, 2017

Download as pdf or txt
Download as pdf or txt
You are on page 1of 5

Solutions to Assignment-3

September 19, 2017

1. Let (X, d) be a metric space, and let Y ⊂ X be a metric subspace with the induced metric dY . Let
E ⊂Y.
(a) Show that a set U ⊂ Y is open in Y if and only if there is a subset V ⊂ X open in X such that
U = V ∩ Y . As an example, consider X = R, Y = [0, 1]. Then U = [0, 1/2) is an open subset
of Y with the induced metric. In this case we can take V = (−1, 1/2). Then V is open in R and
U =Y ∩V.

Solution: We will denote the balls of radius r around p ∈ Y with respect to metrics dY and d
by BrY (p) and BrX (p) respectively. That is

BrY (p) = {y ∈ Y | d(p, y) < r}, BrX (p) = {x ∈ X | d(p, x) < r}.

The key observation is that


BrY (p) = BrX (p) ∩ Y.
As an illustration take X = R and Y = Q. Then B1Q (0) consists of all the rationals in the
interval (−1, 1) while B1R (0) is the whole interval (−1, 1) and clearly B1Q (0) = B1R (0) ∩ Q.
Coming back to the problem, suppose U ⊂ Y is open (with respect to the subspace metric dY ).
Then for each p ∈ U , there exists an rp > 0 such that BrYp (p) ⊂ U. So we can write

U = ∪p∈U BrYp (p).

Let V = ∪p∈U BrXp (p). Then clearly V is open in X since arbitrary union of open sets is open,
and by the above property of balls, it is clear that U = V ∩ Y .
For the converse, suppose U = V ∩ Y where V is open in X. Let p ∈ U . Then p ∈ V , and since
V is open in X, there exists rp > 0 such that BrXp (p) ⊂ V . But then BrYp (p) = BrXp (p) ∩ Y ⊂ U .
This shows that for any p ∈ U we have a ball BrYp (p) ⊂ U , and so U is open in Y .

(b) Show that E is compact subset of Y (with respect to the metric dY ) if and only if it is a compact
subset of X (with respect to the metric d).

Solution: Suppose E ⊂ Y ⊂ X is a compact subset of Y (with respect to dY ). Let {Vα } be


a cover of E by open subsets of X. By the above part, Uα = Vα ∩ Y will be a cover of E by
open subsets of (Y, dY ). Since E is compact with the subspace metric, there exists α1 , · · · αN
such that
E ⊂ ∪N N
k=1 Uαk ⊂ ∪k=1 Vαk .

So we have managed to extract a finite sub-cover of {Vα }. This shows that E is compact in
(X, d). The converse is also similar.

1
(c) Show that E is a connected subset of Y (with respect to the metric dY ) if and only if it is a connected
subset of X (with respect to the metric d).

Solution: Again we show one direction leaving the converse as an excerise. Suppose E ⊂ Y ⊂
X is a connected subset of (Y, dY ) but not (X, d). Then there exists non-empty subsets A and
X X
B of X such that E = A ∪ B but A ∩ B = B ∩ A = φ. Here we are taking closures with
resect to the metric d.
Y Y
Claim. A ∩ B = B ∩ A = φ.
Y
Clearly A ∩ B = φ. Suppose p ∈ A ∩ B. Then p is a limit point of A (in the metric dY ). So
for any r > 0, BrY (p) ∩ A 6= φ. Since BrY (p) ⊂ BrX (p), this shows that BrX (p) ∩ A 6= A for any
r > 0. This shows that p is a limit of A even in the metric d. This is a contradiction since
X
A ∩ B = φ. This completes the proof of the claim.
But then we have written E = A ∪ B where A and B are non-empty separated sets with respect
to the metric dY , contradicting the fact that E is connected with respect to dY . Hence E must
be connected with respect to the metric d too.

2. A subset E ⊂ Rn is called convex if for any two points p, q ∈ E, the straight line
l(t) = (1 − t)p + tq, t ∈ [0, 1]
joining the two points is contained completely in E. Show that any convex set is connected. Hint.
Argue by contradiction.

Solution: If not, then we can write E = A ∪ B, where A and B are non-empty and separated, that
is, A, B 6= Φ, and A ∩ B = A ∩ B = Φ. Recall that this also means that A and B are both open and
closed in E.
Choose p ∈ A and q ∈ B. Since E is convex the straight line l joining them is contained in E.
Intuitively, there will be a first point where the line exits A and enters B. This point will lie in
A ∩ B, a contradiction. More rigorously, let

T = sup{t | l(t) ∈ A for all s ≤ t}.

Then T is the maximum time such that the line is always in A for any time smaller than T .
Claim-1. l(T ) ∈
/ A.
Proof. Now suppose l(T ) ∈ A. Since A is open in E, there is an ε > 0 such that

{x ∈ E | |x − l(T )| < ε} ⊂ A.

Now note that

|l(s) − l(T )| = |(T − s)p + (s − T )q| ≤ |s − T |||p| + |s − T ||q| = |s − T |(|p| + |q|).


ε
So if s ∈ [T, T + |p|+|q| ), then |s − T | < ε/(|p| + |q|), and so

|l(s) − l(T )| < ε.


ε
By our choice of ε and T , this shows that l(s) ∈ A for all s ∈ [0, T + |p|+|q| ), contradicting the
maximality of T . This proves the claim.
Now, since E = A ∪ B, the claim implies that l(T ) ∈ B. In particular T > 0, since l(0) = p ∈ A.
Claim-2. l(T ) is a limit point of A.

2
Proof. Let ε > 0. We need to show that there is some x ∈ Bε (l(T )) ∩ A. Now let
ε
s=T − .
2(|p| + |q|)

Then by the estimate above,


ε
|l(s) − l(T )| ≤ < ε,
2
and so l(s) ∈ Bε (l(T )). But by definition of T , since s < T , automatically l(s) ∈ A, hence proving
the claim.
So we have now shown that l(T ) ∈ A ∩ B, a contradiction.

3. A metric space (X, d) is called separable is it has a countable dense subset. A collection of open sets
{Uα } is called a basis for X if for any p ∈ X and any open set G containing p, p ∈ Uα ⊂ G for some
α ∈ I. The basis is said to be countable if the indexing set I is countable.
(a) Show that Rn is countable. Hint. Q is dense in R.

Solution: Let Qn ⊂ Rn be the set of all points with all rational coordinates. Then Qn is dense
in Rn , and is countable. So Rn is separable.

(b) Prove that a metric is separable if and only if it has a countable basis of open sets. Hint. One
direction is not hard (which one?). For the other direction, think of smaller and smaller balls of
rational radii.

Solution: Proof of ⇐= . Suppose (X, d) has a countable basis {Uk }. We can assume without
loss of generality that none of the basis elements are identical open sets. Pick a point pk ∈ Uk .
Let P be the collection of points {pk }. Note that unless X is finite (in which case the proof
is trivial anyway), we can always pick points which are not repeated. We claim that X = P .
To see this, let x ∈ X \ P . We have to show that x is a limit point of P . Let G be any open
set containing x. Then by definition of basis, there is a k such that x ∈ Uk ⊂ G. But then
pk ∈ G ∩ P . So every open set around x intersects P at a point different from x itself (since
x∈ / P ). This shows that x is a limit point of P .
Proof of =⇒ . Let S = {x1 , x2 , · · · } be the countable dense subset, and let Un,j = B1/n (xj ).
We claim that {Un,j } forms a basis, and since there are clearly countable number of sets, this
completes the proof. To see that it forms a basis, let x ∈ X and G be an open set containing
x. Since G is open, there is an integer N such that the ball B4/N (x) ⊂ G. Since S is dense,
xj ∈ B1/N (x) for some j. Choose an integer N/2 < M < N Then we claim that

x ∈ B1/M (xj ) ⊂ B4/N (x).

The first inclusion follows since M < N and d(x, xj ) < 1/N < 1/M . For the second inclusion,
let y ∈ B1/M (xj ). Then

1 1 3
d(x, y) ≤ d(x, xj ) + d(y, xj ) < + < .
N M N
In particular,
x ∈ UM,j ⊂ G,
completing the proof.

(c) Prove that every compact metric space is separable. Hint. First, cover the metric space by balls of

3
radius 1. By compactness only a finitely many such balls are needed (see problem 1). Continue to
do this for balls of radius 1/n for larger and larger integers. Does this give you a countable dense
subset?

Solution: Let X be a compact metric space. By problem 1, for each integer n there is a
number K(n) and points xn,1 , · · · , xn,K(n) such that
K(n)
X ⊂ ∪k=1 B1/n (xn,k ).

We let Un,k = B1/n (xn,k ).


Claim. S := {xn,k } is a dense subset. Since it is clearly countable, this will prove that X is
separable.
Proof. We need to show that for any x ∈ X and any open set G, G ∩ S is non-empty. Since
G is open, there exists an integer N such that B1/N (x) ⊂ G. But since the collection of balls
K(N )
{B1/N (xN,k )}k=1 is a cover for X, there is some k such that x ∈ B1/N (xN,k ). But then
trivially xN,k ∈ B1/N (x) ⊂ G, and so G ∩ S is non-empty. This completes the proof of the
claim.

4. The aim of this exercise is to complete the proof that compactness and limit point compactness are
equivalent. Let (X, d) be a limit point compact metric space.
(a) Show for every δ > 0, X can be covered by finitely many balls of radius δ. (Note that this is easy
for a set already known to be compact; see problem 4 from the previous assignment).

Solution: Pick any point x1 . Then pick x2 such that d(x2 , x1 ) ≥ δ. If there is no such point
then already X = Bδ (x1 ) and the claim is proved with N = 1. Now inductively, having picked
x1 , x2 , · · · , xn−1 pick an xn such that d(xn , xj ) > δ for all j = 1, · · · n − 1. We claim that the
process terminates in a finitely many steps, thus proving the claim. If not, then we have a
sequence {xn }. Since X is limit point compact, there is a limit point p ∈ X. So there is an
infintely many terms of the sequence in the ball Bδ/2 (p). If xn and xm are two such points,
then d(xn , xm ) ≤ d(xn , p) + d(xm , p) ≤ δ/2 + δ/2 = δ contradicting the fact that d(xn , xm ) > δ.
So the claim is proved.

(b) If Fn is a collection of non-empty closed subsets of X such that Fn+1 ⊂ Fn for all n, then show
that ∩∞n=1 Fn is non-empty.

Solution: Choose points xn ∈ Fn . If the range of the sequence {xn } is finite, then clearly
there is a point, say xN , which is in infinitely many of the sets Fn . Then, since the sets are
decreasing, clearly xN will lie in the common intersection. So suppose the range {xn } is infinite.
Then since X is limit point compact, there is a limit point p.
Claim. p ∈ ∩∞ n=1 Fn .
Proof. If not, then there exists and N such that p ∈ / FN . Since FN is closed, there is a ball
Br (p) such that Br (p) ∩ FN = φ. Since Fn ⊂ FN for all n > N , clearly Br (p) ∩ Fn = φ for all
n > N . On the other hand, since p is a limit point, Br (p) ∩ {xn } has infinitely many points.
In particular there is an n > N such that xn ∈ Br (p). But xn ∈ Fn which is a contradiction
since Br (p) ∩ Fn = φ.

(c) Prove that limit point compactness implies compactness.

Solution: The proof is similar to the proof that closed and bounded sets in Rn are compact.
We proceed by contradiction. Suppose there is an open cover {Gα } such that no finite sub

4
collection covers X. By the proof of part (b) there exists a finite collection of points x1 , · · · xN
such that X ⊂ ∪N j=1 B1 (xj ). At least one of the balls cannot be covered by a finite sub collection
of {Gα }. Label this ball B1 . Clearly B 1 is also limit point compact (Why?), and so we can
cover B1 with finitely many balls of radius 1/2 and pick a ball which cannot be covered by a
finite sub-collection from {Gα }. We continue and obtain a sequence of balls {Bj } such that

• Bj+1 ⊂ Bj for all j.


• The radius of Bj is 1/j.
• No finite sub collection of {Gα } covers Bj . In particular, no Bj is contained in any of the
Gα s.

Now, pick a point xj ∈ Bj . If the range {xj } is finite, then one of the points, say x1 , belongs
to infinitely many of the balls Bj . If x1 ∈ Gα , then since Gα is open, there is a j big enough
so that x1 ∈ Bj ⊂ Gα . But this contradicts property 3 above.
If the range {xj } is infinite, then by limit point compactness, there exists a limit point p ∈ X.
Let α such that p ∈ Gα . Since Gα is open there is an r > 0 such that Br (p) ⊂ Gα . Moreover,
since p is a limit point of {xj }, there is subsequence {xjk } such that xjk ∈ Br/2 (p). Choose jk
big enough such that 1/jk < r/2. Then by triangle inequality, Bjk ⊂ Br (p) ⊂ Gα contradicting
property 3.

You might also like

pFad - Phonifier reborn

Pfad - The Proxy pFad of © 2024 Garber Painting. All rights reserved.

Note: This service is not intended for secure transactions such as banking, social media, email, or purchasing. Use at your own risk. We assume no liability whatsoever for broken pages.


Alternative Proxies:

Alternative Proxy

pFad Proxy

pFad v3 Proxy

pFad v4 Proxy